LSAT and Law School Admissions Forum

Get expert LSAT preparation and law school admissions advice from PowerScore Test Preparation.

 Administrator
PowerScore Staff
  • PowerScore Staff
  • Posts: 8917
  • Joined: Feb 02, 2011
|
#33111
Complete Question Explanation

Must Be True—Formal Logic. The correct answer choice is (C)

The facts in this stimulus describe how some physicians view the marketing practices of drug companies, which often send gifts to physicians. A recent survey asked physicians whether they believed this practice influenced either themselves or other physicians when prescribing drugs. Most physicians reportedly responded that they themselves are not influenced by drug companies’ gifts. However, in response to the same survey, most physicians also reported their belief that most other physicians’ prescription choices are influenced by drug companies’ gifts.

Here are the facts of the stimulus, broken down for review:
  • Fact: ..... In marketing their products, the drug companies often send gifts to physicians.

    Fact: ..... According to a recent survey, most physicians believe their own choices when prescribing drugs are not ..... ..... influenced by drug companies’ gifts.
    • Oct 13_LR1_#22.png
    Fact: ..... According to the same survey, the majority of physicians believe that most other physicians’ prescription ..... ..... choices are influenced by drug companies’ gifts.
    • Oct 13_LR1_#22_diagram_2.png
The task in this Must Be True question is to select the answer choice that presents either an inference permitted by a combination of the facts in the stimulus, or a restatement of one of the facts. Here, the facts permit an inference. The two survey results discussed in the stimulus contain beliefs that are mutually exclusive. The belief held by most physicians that their own choices when prescribing drugs are not affected by the drug companies’ gifts cannot be accurate, if the other belief held by the majority of physicians, that most other physicians’ choices are affected by such gifts, is accurate.

To make the conflict between these two beliefs more clear, imagine that there are 100 physicians in the world. According to the second fact described above, at least 51 of those 100 physicians believe that their own choices are not influenced by the drug companies’ gifts. But, according to the third fact, at least 51 of the 100 physicians each believe that the prescription choices of at least 50 of the remaining 99 physicians are influenced by the gifts.

Your prephrase is that both beliefs indicated by the survey cannot be accurate. Either some of the physicians who indicate that their prescription decisions are not influenced by the drug companies’ gifts are wrong about what influences them, or some of the physicians who believe that most other physicians are influenced by the gifts are overestimating the influence of the gifts on others.

Answer choice (A): The facts in the stimulus do not permit an inference about physicians who do not accept gifts from drug companies.

Answer choice (B): While the stimulus provided survey results reflecting two beliefs held by physicians, neither of those beliefs dealt with what the drug companies should do.

Answer choice (C): This is the correct answer choice. As described above, the facts in the stimulus presented two mutually exclusive beliefs apparently held by physicians. At least one (i.e., “some”) physician is either mistaken about the first belief expressed or is mistaken about the second. It cannot be the case that both beliefs reported in the survey results are accurate.

Answer choice (D): The stimulus provided no information regarding the beliefs of physicians who admit that their own choices when prescribing drugs are influenced by drug companies’ gifts.

Answer choice (E): As with answer choice (D), the facts in the stimulus do not permit an inference regarding the beliefs of physicians who admit to being influenced by the drug companies’ gifts.
You do not have the required permissions to view the files attached to this post.
 15veries
  • Posts: 113
  • Joined: Sep 25, 2016
|
#29583
Why is C correct?
I thought since it says "most...are not influenced" and "majority...by such gifts" there are AT LEAST some people think both that their own choices when prescribing drugs are not influenced by drug companies gifts AND that most other physicians prescription choices are influenced by such gifts" This does not seem contradict...and it does not seem exactly the same as what C says.
What did I miss?
 Emily Haney-Caron
PowerScore Staff
  • PowerScore Staff
  • Posts: 577
  • Joined: Jan 12, 2012
|
#29640
Hi 15veries,

Most (more than 50%) believe they are not influenced by gifts, and most (more than 50%) believe most other doctors are influenced. Even if these numbers are as small as possible, that means some doctors have to be wrong. For example, let's say they surveyed 100 doctors. 51 doctors said they are not influenced by gifts, 49 said they are. 51 doctors said other doctors are influenced by gifts, 49 said they aren't. Only 49 doctors admit to being influenced, but 51 doctors think gifts influence other doctors. Somebody is wrong. Therefore, C.

Does that help break it down a bit?
 15veries
  • Posts: 113
  • Joined: Sep 25, 2016
|
#29707
Yes thank you,
So I guess we should not use the ben diagram here right?
(Most+Most: infer some)
User avatar
 Dave Killoran
PowerScore Staff
  • PowerScore Staff
  • Posts: 5852
  • Joined: Mar 25, 2011
|
#29710
Hi 15veries,

Thanks for the reply! Well, we don't really use Venn diagrams (here's why: The Limitations of Venn Diagramming) but that doesn't mean we can't understand this in formal logic terms. And on that mark, you appear to have understood this perfectly since you correctly noted the "Most+Most: infer some" relationship that is present here. That can be diagrammed if needed, although I feel like in most cases it doesn't need to be diagrammed since it is so clear.

Please let me know if that helps. Thanks!
 htngo12
  • Posts: 40
  • Joined: May 19, 2016
|
#33673
I was working on the formal logic in the Logic Bible last week and learned about "Most are not" A (most)-> not B.

In the stim, it presented the first survey as "most physicians believe... are not influenced..." I phrased it as "Most are not"
with scale (0 not influenced to 100 influenced) the possibly of (49/99) saying they are not influenced by gifts while poss (51/100) said they were.

With the second part, I was able phrase it as " Most" with scale (0 not influenced to 100 influenced) so (51/100) say other docs are influenced by gifts while poss (49/99) are not.

Since there is an overlap of both the scales, there must be an error in their beliefs.

For first survey, "Most are not" can be inferred down to "Some are not influenced."

For the second survey, "Most" can be inferred down to "Some."

I'm not sure I'm looking at this correctly, based on what I learned in LR bible versus the previous posts.
 Kristina Moen
PowerScore Staff
  • PowerScore Staff
  • Posts: 230
  • Joined: Nov 17, 2016
|
#33696
Hi ht,

Careful with changing "most" to "some." On this test, they do not mean the same thing. One a scale of 100, Some = (1 - 100) whereas most = (51 - 100). Here, if the stimulus said "Some physicians do not believe they are influenced" and "Some physicians believe most others are influenced," then you would have not bean able to select answer choice (C).

The first post in this thread shows you how we would diagram the two statements.

Image
Image

The "m" indicates "most." This differentiates it from conditional reasoning, where ALL of the people in the left category are in the right category. Here, most people in the left category are also in the right category.

Have you ever heard the joke "Most people think they are a better-then-average driver?" This is a similar idea. Keep working on formal logic questions - sounds like you are getting it!
 htngo12
  • Posts: 40
  • Joined: May 19, 2016
|
#33792
Thanks! I think I get it now.

I was confusing a conditional logic stimulus vs inference one.

Get the most out of your LSAT Prep Plus subscription.

Analyze and track your performance with our Testing and Analytics Package.